LSAT and Law School Admissions Forum

Get expert LSAT preparation and law school admissions advice from PowerScore Test Preparation.

 Administrator
PowerScore Staff
  • PowerScore Staff
  • Posts: 8916
  • Joined: Feb 02, 2011
|
#43402
Please post your questions below! Thank you!
 gab1234
  • Posts: 12
  • Joined: Jul 28, 2020
|
#79460
I wanted to answer with E so badly. I didn't because I thought sharing information about the curry would have been too far of a stretch. :oops: Any tips to avoid this type of error in the future?
 Adam Tyson
PowerScore Staff
  • PowerScore Staff
  • Posts: 5153
  • Joined: Apr 14, 2011
|
#79498
I'm not sure what you mean about sharing information, gab1234, but in a Strengthen question you absolutely need to accept the new information in the answer as 100% true. It's not any kind of a stretch, but a question of "does this help?" If it's true that high curry consumption correlates with low incidence of Alzheimer's, does that give you more confidence in the conclusion that turmeric helps prevent the disease? That's all you need to worry about, and stronger answers tend to be better answers. These aren't like Must Be True questions, where relying on new information is forbidden. Embrace the new facts and see if they do you any good! The same is true for Justify the Conclusion, Resolve the Paradox, and Weaken questions - new information is necessary, and you should accept every answer choice as true.
 kells__w
  • Posts: 10
  • Joined: Mar 29, 2021
|
#87304
Hi,
can someone explain what's wrong with answer C and why E is a better answer. I didn't pick E because the stim said turmeric which contains the compound reducing the accumulation of amyloid proteins is commonly found in curries which I took to mean that not all of them contain tumeric. So the area's consuming the highest per capita rates of curry consumption having the lowest incidence of Alzhimer's mentioned in E could have been composed of areas that predominantly ate curries without tumeric thereby weakening the conclusion in the stimulus.
Thanks in advance!
 Robert Carroll
PowerScore Staff
  • PowerScore Staff
  • Posts: 1787
  • Joined: Dec 06, 2013
|
#87381
Kells,

Answer choice (C) definitely weakens the argument. I'll illustrate. I just looked up information about the distribution of ages throughout the world. Apparently, according to Wikipedia, in the United Arab Emirates, only 1.1% of the entire population is 65 or older. Alzheimer's is so rare among people under 65 that I found it very difficult to even get numbers. So, without knowing anything else that what I just told you, I'd predict that the United Arab Emirates has a very low incidence of Alzheimer's. If I could find anything that people in the UAE do more than people in other countries, I could make the claim that that activity is correlated with a decreased risk of Alzheimer's. It looks like there's a deeper connection between that country is doing and Alzheimer's, but that's only true because the UAE has a young population.

So, what if India also has a relatively low proportion of people 65 or older? Then that would do a lot to explain the low incidence of Alzheimer's, not their consumption of turmeric. And that's what answer choice (C) is telling me.

For answer choice (E), you pointed out that it's compatible with the answer that curry consumption and low incidence of Alzheimer's are not perfectly correlated. True, but the answer choice tightens the connection. The answer choice allows room for the conclusion to be false, but the amount of room for the conclusion to be false isn't expanded. It's been contracted. The fact that answer choice (E) allows the conclusion to be false doesn't mean it weakens it - the conclusion now has LESS of a way to be wrong, not more. And, for a Strengthen question, all we have to do is make the argument's conclusion more likely to some degree.

Robert Carroll

Get the most out of your LSAT Prep Plus subscription.

Analyze and track your performance with our Testing and Analytics Package.